Đến nội dung

Hình ảnh

Topic BẤT ĐẲNG THỨC ôn thi vào lớp 10 THPT 2017 - 2018

bất đẳng thức am-gm cauchy bunyakovski minskovski schwarz holder thcs

  • Please log in to reply
Chủ đề này có 299 trả lời

#41
TrBaoChis

TrBaoChis

    Hạ sĩ

  • Banned
  • 81 Bài viết

Lời giải bài 7:

                   Theo giả thiết ta có $3b\geq (\frac{b^{2}}{2})+(\frac{b^{2}}{4}+c^{2})+(\frac{b^{2}}{4}+c^{2})$

                   Mà $\frac{b^{2}}{4}+c^{2}\geq bc$ và $\frac{b^{2}}{4}+a^{2}\geq ab$

                   Suy ra $3b\geq \frac{b^{2}}{2}+bc+ab$

                   $\Rightarrow 3 \geq \frac{b}{2}+a+c$ $(1)$

 

     Ta có:

$P=\frac{1}{(a+1)^{2}}+\frac{4}{(b+2)^{2}}+\frac{4}{(c+3)^{2}}+\frac{4}{(c+3)^{2}}$

           $ \geq \frac{1}{4}(\frac{1}{a+1}+\frac{2}{b+2}+\frac{2}{c+3}+\frac{2}{c+3})^{2}$

           $=\frac{1}{4}(\frac{1}{a+1}+\frac{1}{\frac{b}{2}+1}+\frac{4}{c+3})^{2}$

           $\geq \frac{1}{4}(\frac{(1+1+2)^{2}}{a+1+\frac{b}{2}+1+c+3})^{2}$

$\Rightarrow P \geq \frac{1}{4}(\frac{(1+1+2)^{2}}{a+1+\frac{b}{2}+1+c+3})^{2}$  $(2)$

Sử dụng $(1)$ thì ta được $\frac{1}{4}(\frac{(1+1+2)^{2}}{a+1+\frac{b}{2}+1+c+3})^{2} \geq \frac{1}{4}(\frac{4^{2}}{3+5})^{2}= 1$  $(3)$

 Từ $(2)$ và $(3)$ thì ta được $P \geq 1$

Đẳng thức xảy ra khi $a=c=1, b=2$

 

? mỗi b dương thôi bạn


Bài viết đã được chỉnh sửa nội dung bởi TrBaoChis: 19-04-2017 - 15:38


#42
Nguyen Xuan Hieu

Nguyen Xuan Hieu

    Binh nhất

  • Thành viên mới
  • 39 Bài viết

Lời giải bài 17:

Ta có các bất đẳng thức sau:

         $3=a+b+c\geq 3\sqrt[3]{abc} \Rightarrow abc\leq 1 $

         

          $9=(a+b+c)^{2}\geq 3(ab+bc+ca) \Rightarrow ab+bc+ca\leq 3$

       

          $3\sqrt[3]{a^{2}b^{2}}(3\sqrt[3]{c^{2}}+1)= 9\sqrt[3]{a^{2}b^{2}c^{2}}+3\sqrt[3]{a^{2}b^{2}} \leq [3(a^{2}b^{2}c^{2}+1+1)]+                   (a^{2}b^{2}+1+1)$     $(AM-GM)$

 

           $\Rightarrow    3\sqrt[3]{a^{2}b^{2}}(3\sqrt[3]{c^{2}}+1)-8 \leq 3a^{2}b^{2}c^{2}+a^{2}b^{2}$

 

           $\Rightarrow \frac{a^{2}+1}{3\sqrt[3]{a^{2}b^{2}}(3\sqrt[3]{c^{2}}+1)-8} \geq \frac{a^{2}+1}{3a^{2}b^{2}c^{2}+a^{2}b^{2}}$

 

           $\Rightarrow \sum_{cyc}\frac{a^{2}+1}{3\sqrt[3]{a^{2}b^{2}}(3\sqrt[3]{c^{2}}+1)-8} \geq \sum_{cyc}\frac{a^{2}+1}         {3a^{2}b^{2}c^{2}+a^{2}b^{2}}$

 

Mà      $ \sum_{cyc}\frac{a^{2}+1}{3a^{2}b^{2}c^{2}+a^{2}b^{2}} \geq \sum_{cyc}\frac{(a+1)^{2}}{6a^{2}b^{2}c^{2}+2a^{2}b^{2}} \geq$

           $\frac{(a+b+c+1+1+1)^{2}}{18a^{2}b^{2}c^{2}+2(a^{2}b^{2}+b^{2}c^{2}+c^{2}a^{2})}= \frac{18}{   9a^{2}b^{2}c^{2}+a^{2}b^{2}+b^{2}c^{2}+c^{2}a^{2}}$  $(1)$

Mặt khác 

                       $9a^{2}b^{2}c^{2}+a^{2}b^{2}+b^{2}c^{2}+c^{2}a^{2} \leq 9abc.1 + a^{2}b^{2}+b^{2}c^{2}+c^{2}a^{2}$

                        $= 2abc(a+b+c)+ a^{2}b^{2}+b^{2}c^{2}+c^{2}a^{2} + 3abc = (ab+bc+ca)^{2}+3abc \leq 3^{2}+3= 12$

                       $\Rightarrow 9a^{2}b^{2}c^{2}+a^{2}b^{2}+b^{2}c^{2}+c^{2}a^{2} \leq 12$  $(2)$

Từ $(1)$ và $(2)$ thì $\sum_{cyc}\frac{a^{2}+1}{3a^{2}b^{2}c^{2}+a^{2}b^{2}} \geq \frac{3}{2}$

                                $\Rightarrow \sum_{cyc}\frac{a^{2}+1}{3\sqrt[3]{a^{2}b^{2}}(3\sqrt[3]{c^{2}}+1)-8} \geq \frac{3}{2}$  $(Q.E.D)$

Cách khác ngắn gọn hơn:
Điều phải chứng minh:
$\sum_{cyc} \dfrac{2(a^2+1)}{3\sqrt[3]{a^2b^2}(1+3\sqrt[3]{c^2})-8}\geq 3$.
Ta có:
$\sum_{cyc} \dfrac{2(a^2+1)}{3\sqrt[3]{a^2b^2}(1+3\sqrt[3]{c^2})-8}
\\=\sum_{cyc} \dfrac{a^2+a^2+1+1}{3\sqrt[3]{a^2b^2}+9\sqrt[3]{a^2b^2c^2}-8}
\\\geq \sum_{cyc} \dfrac{a^2+2a+1}{a+b+ab+3(ab+bc+ca)-8}
\\\geq \sum_{cyc} \dfrac{a^2+2a+1}{a+b+ab+(a+b+c)^2-8}
\\=\sum_{cyc} \dfrac{(a+1)^2}{(a+1)(b+1)}
\\=\sum_{cyc} \dfrac{(a+1)}{(b+1)}
\\\geq 3$.

 



#43
AnhTran2911

AnhTran2911

    Thượng sĩ

  • Thành viên
  • 230 Bài viết

Các bạn chú ý lần sau đăng bài xin trích rõ nguồn nhé.

 

Vâng để em nhắc Hưng sửa lại cho đúng. Đôi khi nhầm lẫn là chuyện bình thường. Bài toán đề xuất tiếp theo:

 

Bài 18: (Vasile Cirtoaje)

Cho $x, y, z > 0$ ; $x+y+z=3$. Chứng minh rằng:

$\sum_{cyc} \frac{x}{xy+1} \geq \frac{3}{2}$ 

Áp dụng BĐT CS kết hợp bổ đề Vasile quen thuộc ta có:

$\sum_{cyc} \frac{x}{xy+1} \geq \frac{A}{B}$ trong đó :

A= $\sum\ (\sqrt{x^{3}}+\sqrt{y^{3}}+\sqrt{z^{3}})^{2}$

B= $\frac{(x^{2}+y^{2}+z^{2})^{2}}{3} + x^{2}+y^{2}+z^{2}$

Áp dụng BDT HOLDER ta có:

$3A\geq\ (x^{2}+y^{2}+z^{2})^{3}$.

Để đơn giản hóa ta đặt: $x^2+y^2+z^2$ = t ( t>=3) lúc này BĐT đã cho trở thành:

$\frac{t^2}{t+3}\geq\frac{3}{2}$. Hiển nhiên đúng vì nó tương đương với: (2t+3)(t-3)>=0 . ĐPCM.


Bài viết đã được chỉnh sửa nội dung bởi AnhTran2911: 20-04-2017 - 16:28

        AQ02

                                 


#44
NHoang1608

NHoang1608

    Sĩ quan

  • Thành viên
  • 375 Bài viết

Đề xuất bài tiếp theo:

 

Bài 19 (Tạp chí TTT):

Cho các số thực không âm thỏa mãn $a^{2}+b^{2}+c^{2} \not = 0$

Tìm giá trị nhỏ nhất của biểu thức

                                                      $P=\frac{\sqrt{a+2b+3c}}{\sqrt{2a+2b+3c}+\sqrt{a+5b+3c}+\sqrt{a+2b+7c}}$

 

Bài 20 (Tạp chí TTT):

Cho $a,b,c$ là các số thực dương .Chứng minh rằng:

                                                                      $a^{2}+b^{2}+c^{2}\geq \frac{9abc}{a+b+c}+(c-a)^{2}$

 

Bài 21 (Tạp chí AMM):

 Cho các số $a,b,c \in [1;2]$. Chứng minh rằng:

                                                                        $(3a+2b+c)(\frac{1}{a}+\frac{1}{b}+\frac{1}{c})\leq \frac{45}{2}$

Dấu bằng xảy ra khi nào ? 


Bài viết đã được chỉnh sửa nội dung bởi NHoang1608: 20-04-2017 - 21:05

The greatest danger for most of us is not that our aim is too high and we miss it, but that it is too low and we reach it.

----- Michelangelo----


#45
AnhTran2911

AnhTran2911

    Thượng sĩ

  • Thành viên
  • 230 Bài viết

Đề xuất bài tiếp theo:

 

Bài 19 (Tạp chí TTT):

Cho các số thực không âm thỏa mãn $a^{2}+b^{2}+c^{2} \not = 0$

Tìm giá trị nhỏ nhất của biểu thức

                                                             $P=\frac{\sqrt{a+2b+3c}}{\sqrt{2a+2b+3c}+\sqrt{a+5b+3c}+\sqrt{a+2b+7c}}$

 

Bài 20 (Tạp chí TTT):

Cho $a,b,c$ là các số thực dương .Chứng minh rằng:

                                                              $a^{2}+b^{2}+c^{2}\geq \frac{9abc}{a+b+c}+(c-a)^{2}$

 

Bài 21 (Tạp chí AMM):

 Cho các số $a,b,c \in [1;2]$. Chứng minh rằng:

                                                              $(3a+2b+c)(\frac{1}{a}+\frac{1}{b}+\frac{1}{c})\leq \frac{45}{2}$

Dấu bằng xảy ra khi nào ? 

2 bài đầu có trong mục chữa bài ở báo .Xin góp ý cho bài toàn cuối:

Ta có Bổ đề quen thuộc sau: Với $a,b,c \in [1;2]$. CM: $(a+b+c)(\frac{1}{a}+\frac{1}{b}+\frac{1}{c})\leq10$

Bổ đề trên các bạn tham khảo ở NCPT toán 9 tập 1.

Áp dụng bất đẳng thức trên ta quy bài toán về cm : $(2a+b)(\frac{1}{a}+\frac{1}{b}+\frac{1}{c})\leq\frac{25}{2}$

 

Khai triển và sử dụng gt c>=1 ta có: $\frac{2a}{b} +\frac{b}{a} +2a+b \leq\frac{19}{2}$

BĐT này tương đương với : $(4+4b)a^{2} +a(2b^{2}-19b) +2b^{2}\leq 0$

Nhân thấy đây là tam thức d2, tham số b. Hệ số cao nhất dương . a thuộc khoảng [1,2 ] nên theo định lý về hàm cực trị biên thì :

$f(a)\leq max({f(1);f(2))}$ . Đến đây thì đơn giản rồi ta xét BĐT trên tại a=1 và a=2 để từ đó CM vs BĐT 1 biến b với ĐK b thuộc [1,2]. CM cái này không mấy khó khăn, các bạn hãy thử xem.


Bài viết đã được chỉnh sửa nội dung bởi AnhTran2911: 20-04-2017 - 17:46

        AQ02

                                 


#46
Nguyenphuctang

Nguyenphuctang

    Sĩ quan

  • Banned
  • 499 Bài viết

Bài 22: (Lê Việt Hưng boxmath - Mr Coopper)

Cho a,b,c là các số thực dương. Chứng minh rằng:
\[{\sum {\left( {\frac{{{a^2} - ab + {b^2}}}{{{a^2} + ab + {b^2}}}} \right)} ^2} \ge \frac{1}{3} + \frac{2}{9}\frac{{{{\left( {{a^2} + {b^2} + {c^2} - ab - bc - ca} \right)}^2}}}{{{a^4} + {b^4} + {c^4}}}\]
 
Bài 23: (Lil.Tee boxmath - Tăng Hải Tuân)
Cho $a, b, c$ là các số thực dương có tổng là $3$. Chứng minh rằng: $$\dfrac{a}{b^2+\sqrt{c}}+ \dfrac{b}{c^2+\sqrt{a}}+ \dfrac{c}{a^2+\sqrt{b}} \geq \dfrac{3}{3-\sqrt[3]{abc}}$$

Bài viết đã được chỉnh sửa nội dung bởi Nguyenphuctang: 21-04-2017 - 16:37


#47
tuaneee111

tuaneee111

    Trung sĩ

  • Thành viên
  • 174 Bài viết

Bài 22:

Ta có:

\[\frac{{2{{\left( {\sum\limits_{cyc} {{{\left( {a - b} \right)}^2}} } \right)}^2}}}{{9\left( {{a^4} + {b^4} + {c^4}} \right)}} = \frac{{{{\left( {\sum\limits_{cyc} {{{\left( {a - b} \right)}^2}} } \right)}^2}}}{{18\left( {{a^4} + {b^4} + {c^4}} \right)}} \le \frac{{\sum\limits_{cyc} {{{\left( {a - b} \right)}^4}} }}{{6\left( {{a^4} + {b^4} + {c^4}} \right)}}\]

Khi đó ta có:

\[VT - \frac{{\sum\limits_{cyc} {{{\left( {a - b} \right)}^4}} }}{{6\left( {{a^4} + {b^4} + {c^4}} \right)}} = \sum\limits_{cyc} {{{\left( {a - b} \right)}^2}\left( {\frac{{2\left( {4{a^2} - ab + 4{b^2}} \right)}}{{9{{\left( {{a^2} + ab + {b^2}} \right)}^2}}} - \frac{{{{\left( {a - b} \right)}^2}}}{{6\left( {{a^4} + {b^4} + {c^4}} \right)}}} \right)}  \ge \sum\limits_{cyc} {{{\left( {a - b} \right)}^2}\left( {\frac{{2\left( {4{a^2} - ab + 4{b^2}} \right)}}{{9{{\left( {{a^2} + ab + {b^2}} \right)}^2}}} - \frac{{{{\left( {a - b} \right)}^2}}}{{6\left( {{a^4} + {b^4}} \right)}}} \right)} \]

Bất đẳng thức đúng nếu ta chứng minh được đánh giá sau đúng:

\[4\left( {4{x^2} - x + 4} \right)\left( {{x^4} + 1} \right) > 3{\left( {x - 1} \right)^2}{\left( {{x^2} + x + 1} \right)^2} \Leftrightarrow 13{x^6} - 4{x^5} + 16{x^4} + 6{x^3} + 16{x^2} - 4x + 13 > 0\]

Dễ thấy đánh giá trên luôn đúng nên có đpcm


$$\boxed{\boxed{I\heartsuit MATHEMATICAL}}$$

Blog của tôi

:luoi: Sức hấp dẫn của toán học mãnh liệt đến nỗi tôi bắt đầu sao nhãng các môn học khác - Sofia Vasilyevna Kovalevskaya :lol:


#48
cristianoronaldo

cristianoronaldo

    Thượng sĩ

  • Thành viên
  • 233 Bài viết

 

Bài 22: (Lê Việt Hưng boxmath - Mr Coopper)

Cho a,b,c là các số thực dương. Chứng minh rằng:
\[{\sum {\left( {\frac{{{a^2} - ab + {b^2}}}{{{a^2} + ab + {b^2}}}} \right)} ^2} \ge \frac{1}{3} + \frac{2}{9}\frac{{{{\left( {{a^2} + {b^2} + {c^2} - ab - bc - ca} \right)}^2}}}{{{a^4} + {b^4} + {c^4}}}\]
 
Bài 23: (Lil.Tee boxmath - Tăng Hải Tuân)
Cho $a, b, c$ là các số thực dương có tổng là $3$. Chứng minh rằng: $$\dfrac{a}{b^2+\sqrt{c}}+ \dfrac{b}{c^2+\sqrt{a}}+ \dfrac{c}{a^2+\sqrt{b}} \geq \dfrac{3}{3-\sqrt[3]{abc}}$$

 

Lời giải câu 23:

Áp dụng bất đẳng thức Cauchy-Schwarz ta có:

$\sum \frac{a}{b^2+\sqrt{c}}\geq \frac{(\sum a)^2}{\sum ab^2+\sum a\sqrt{c}}=\frac{9}{\sum ab^2+\sum a\sqrt{c}}$

Cũng theo bất đẳng thức Cauchy-Schwarz ta có:

$\sum a\sqrt{c}\leq \sqrt{(\sum a)(\sum ab)}\leq \sqrt{\frac{(\sum a)^3}{3}}=3$

Mặt khác sử dụng bổ đề :$\sum ab^2\leq \frac{4}{27}.(\sum a)^3-abc=4-abc$

Như vậy ta cần chứng minh:

$\frac{9}{7-abc}\geq \frac{3}{3-\sqrt[3]{abc}}\Leftrightarrow abc+2\geq 3\sqrt[3]{abc}$(luôn đúng theo AM-GM)

Suy ra đpcm


Bài viết đã được chỉnh sửa nội dung bởi cristianoronaldo: 22-04-2017 - 19:16

Nothing in your eyes


#49
Nguyenphuctang

Nguyenphuctang

    Sĩ quan

  • Banned
  • 499 Bài viết

Bài 24: USA MO 2017 ngày 2.

Cho $a, b, c, d \geq 0; a+b+c+d=4$. Tìm GTNN:

$P= \sum_{cyc} \frac{a}{b^{3}+4} $

 

Các bạn làm đề gốc trước mở rộng mình sẽ đăng sau. :D 



#50
Mr Cooper

Mr Cooper

    Sĩ quan

  • Thành viên
  • 496 Bài viết

Bài 24: USA MO 2017 ngày 2.

Cho $a, b, c, d \geq 0; a+b+c+d=4$. Tìm GTNN:

$P= \sum_{cyc} \frac{a}{b^{3}+4} $

 

Các bạn làm đề gốc trước mở rộng mình sẽ đăng sau. :D

 

$\boxed{6}$

Để ý rằng: $\dfrac{1}{x^3+4}=\dfrac{3-x}{12}+\dfrac{x(x-2)^2(x+1)}{12(x^3+4)}$

$\Rightarrow \sum \dfrac{a}{b^3+4}\ge \dfrac{3(a+b+c+d)-(a+c)(b+d)}{12}\ge \dfrac{2}{3}$
Sử dụng bất đẳng thức $\text{AM-GM}$ ta có: $(a+c)(b+d)\le \dfrac{(a+b+c+d)^2}{4}=4$
Hoàn tất chứng minh

Đẳng thức xảy ra khi: $(a,b,c,d)=(2;2;0;0)$ và các hoán vị tương ứng



#51
Nguyenphuctang

Nguyenphuctang

    Sĩ quan

  • Banned
  • 499 Bài viết

Mình nghĩ 1 số bài sau đây sẽ giúp ích cho các bạn khá nhiều đấy :D 

http://nguyenphuctan...uchy-nguoc.html



#52
Nguyenphuctang

Nguyenphuctang

    Sĩ quan

  • Banned
  • 499 Bài viết

Bài 25: Jack Garfunkel

Cho $a, b, c \geq 0$. Chứng minh rằng:

$$\frac{a^{2}+b^{2}+c^{2}}{ab+bc+ca} + \frac{8abc}{(a+b)(b+c)(c+a) } \geq 2$$

 Mình đăng bài này có mục đích trước hết các bạn hãy giải bài này xong. Sau đó mình sẽ làm trội bài này bằng 1 bổ đề rất rất chặt :D


Bài viết đã được chỉnh sửa nội dung bởi Nguyenphuctang: 26-04-2017 - 22:44


#53
viet9a14124869

viet9a14124869

    Trung úy

  • Thành viên
  • 903 Bài viết

Bài 25: Jack Garfunkel

Cho $a, b, c \geq 0$. Chứng minh rằng:

$$\frac{a^{2}+b^{2}+c^{2}}{ab+bc+ca} + \frac{8abc}{(a+b)(b+c)(c+a) } \geq 2$$

 Mình đăng bài này có mục đích trước hết các bạn hãy giải bài này xong. Sau đó mình sẽ làm trội bài này bằng 1 bổ đề rất rất chặt :D

Do vai trò của a,b,c như nhau nên ta có thể giả sử $a\geq b\geq c\geq 0$ 

Từ đó ta có $(a+b)(b+c)(c+a)\leq 2b(a+c)^2$ ( Cái này là biến đổi tương đương ,các bạn thử tách ra nhé  :icon6: ) 

Bất đẳng thức cần chứng minh trở thành $\frac{a^2+b^2+c^2}{ab+bc+ca}+\frac{4ac}{(a+c)^2}\geq 2\Leftrightarrow \frac{(a^2+c^2-ab-bc)^2}{(ab+bc+ca)(a+c)^2}\geq 0$  ( Đúng ) 

Vậy là ta đã giải quyết xong luôn bài toán tại đây https://diendantoanh...bcabbcca-geq-2/

Dấu bằng xảy ra khi c=0,a=b=1 và các hoán vị

P/s : Mình cũng lờ mờ đoán được ý của Tăng ( đoán thôi nhé  :icon6: ) về bài toán trội của bài này . Đó là :

Ta chứng minh $\frac{a^2+b^2+c^2}{ab+bc+ca}+\frac{8abc}{(a+b)(b+c)(c+a)}\geq 2+\frac{4(a-b)^2(b-c)^2(c-a)^2}{(a+b+c)(ab+bc+ca)(a+b)(b+c)(c+a)}$ 


                                                                    SÓNG BẮT ĐẦU TỪ GIÓ

                                                                    GIÓ BẮT ĐẦU TỪ ĐÂU ?

                                                                    ANH CŨNG KHÔNG BIẾT NỮA 

                                                                    KHI NÀO...? TA YÊU NHAU .


#54
Nguyenphuctang

Nguyenphuctang

    Sĩ quan

  • Banned
  • 499 Bài viết

Làm trội lên :

(Lê Khánh Sỹ, tạp chí toán học rumania)

Cho $a, b, c \geq 0$; $c=min\left \{ a,b,c \right \}$ . Chứng minh rằng:

$$\frac{a^{2}+b^{2}+c^{2}}{ab+bc+ca}+ \frac{8abc}{(a+b)(b+c)(c+a)} \geq 2 + \frac{2c(a-b)^{2}}{3(a+b)(b+c)(c+a)}$$

 

Nếu các bạn có thể làm trội thì xin đưa ra phép chứng minh cho mọi người cùng tham khảo.

Xin mời các bạn chứng minh bổ đề trên trước :D 



#55
Nguyenhuyen_AG

Nguyenhuyen_AG

    Trung úy

  • Thành viên nổi bật 2016
  • 945 Bài viết

Làm trội lên :

(Lê Khánh Sỹ, tạp chí toán học rumania)

Cho $a, b, c \geq 0$; $c=min\left \{ a,b,c \right \}$ . Chứng minh rằng:

$$\frac{a^{2}+b^{2}+c^{2}}{ab+bc+ca}+ \frac{8abc}{(a+b)(b+c)(c+a)} \geq 2 + \frac{2c(a-b)^{2}}{3(a+b)(b+c)(c+a)}$$

 

Ta có

\[\text{VT-VP} = \frac{\left[(ab+2bc+5ca)(a-c)+(2a^2+3ab+3bc)(b-c)\right](a-b)^2+3c^2(a+b)(a-c)(b-c)}{3(a+b)(b+c)(c+a)(ab+bc+ca)} \geqslant 0.\]


Nguyen Van Huyen
Ho Chi Minh City University Of Transport

#56
Mr Cooper

Mr Cooper

    Sĩ quan

  • Thành viên
  • 496 Bài viết

Bài toán làm mạnh một bất đẳng thức quen thuộc

 

$\boxed{\textbf{Bài Toán 26}}$ $\text{[Lê Việt Hưng - Mr Cooper]} $ Chứng minh bất đẳng thức sau đúng với mọi a,b,c. 

\[a^2+b^2+c^2 \ge \sum \dfrac{a(b^2+c^2)}{b+c} \ge ab+bc+ca\]


Bài viết đã được chỉnh sửa nội dung bởi Mr Cooper: 28-04-2017 - 11:49


#57
Mr Cooper

Mr Cooper

    Sĩ quan

  • Thành viên
  • 496 Bài viết

Bài 25: Jack Garfunkel

Cho $a, b, c \geq 0$. Chứng minh rằng:

$$\frac{a^{2}+b^{2}+c^{2}}{ab+bc+ca} + \frac{8abc}{(a+b)(b+c)(c+a) } \geq 2$$

 Mình đăng bài này có mục đích trước hết các bạn hãy giải bài này xong. Sau đó mình sẽ làm trội bài này bằng 1 bổ đề rất rất chặt :D

Có tất cả 6 cách chứng minh cho Bài Toán này. Xem tại đây

 

Nguồn: Lê Khánh Sỹ


Bài viết đã được chỉnh sửa nội dung bởi Mr Cooper: 28-04-2017 - 20:11


#58
Nguyenphuctang

Nguyenphuctang

    Sĩ quan

  • Banned
  • 499 Bài viết

Có tất cả 6 cách chứng minh cho Bài Toán này. Xem tại đây

Ghi nguồn vào nhé em ! Tôn trọng tác giả đi tuy thân nhưng dù sao cũng phải tôn trọng anh ấy!



#59
tuaneee111

tuaneee111

    Trung sĩ

  • Thành viên
  • 174 Bài viết

Bài toán làm mạnh một bất đẳng thức quen thuộc

 

$\boxed{\textbf{Bài Toán 26}}$ $\text{[Lê Việt Hưng - Mr Cooper]} $ Chứng minh bất đẳng thức sau đúng với mọi a,b,c. 

\[a^2+b^2+c^2 \ge \sum \dfrac{a(b^2+c^2)}{b+c} \ge ab+bc+ca\]

Giả sử: \[a = \max \left\{ {a,b,c} \right\}\]

* Chứng minh vế đầu tiên.

Ta có:

\[{a^2} + {b^2} + {c^2} - \sum\limits_{cyc} {\frac{{a\left( {{b^2} + {c^2}} \right)}}{{b + c}}}  = {S_c}{\left( {a - b} \right)^2} + {S_b}{\left( {a - c} \right)^2} + {S_a}{\left( {b - c} \right)^2}\]

Trong đó:

\[{S_c} = \frac{{ab + bc + ca - {c^2}}}{{2\left( {a + c} \right)\left( {b + c} \right)}};\,\,\,{S_a} = \frac{{ab + bc + ca - {a^2}}}{{2\left( {c + a} \right)\left( {b + a} \right)}};\,\,{S_b} = \frac{{ab + bc + ca - {b^2}}}{{2\left( {b + a} \right)\left( {b + c} \right)}}\]

Mặt khác:

\[{a^2}{S_b} + {b^2}{S_a} = \frac{1}{{2\left( {a + b} \right)}}\left( {\frac{{{a^2}\left( {ab + bc + ca - {b^2}} \right)}}{{\left( {b + c} \right)}} + \frac{{{b^2}\left( {ab + bc + ca - {a^2}} \right)}}{{\left( {c + a} \right)}}} \right)\]

\[ \ge \frac{1}{{2\left( {a + b} \right)}}\left( {\frac{{{a^2}\left( {ab + bc + ca - {b^2}} \right)}}{{a + c}} + \frac{{{b^2}\left( {ab + bc + ca - {a^2}} \right)}}{{a + c}}} \right) \ge 0\]

Nên ta có:

\[{a^2} + {b^2} + {c^2} \ge \sum\limits_{cyc} {\frac{{a\left( {{b^2} + {c^2}} \right)}}{{b + c}}} \]

Chứng minh vế thứ 2.

Ta có:

\[\sum\limits_{cyc} {\frac{{a\left( {{b^2} + {c^2}} \right)}}{{b + c}}}  - \sum\limits_{cyc} {ab}  = \sum\limits_{cyc} {\frac{{{c^2}{{\left( {a - b} \right)}^2}}}{{\left( {a + c} \right)\left( {b + c} \right)}}}  \ge 0\]

Vậy bất đẳng thức được chứng minh thành công.


Bài viết đã được chỉnh sửa nội dung bởi tuaneee111: 28-04-2017 - 20:10

$$\boxed{\boxed{I\heartsuit MATHEMATICAL}}$$

Blog của tôi

:luoi: Sức hấp dẫn của toán học mãnh liệt đến nỗi tôi bắt đầu sao nhãng các môn học khác - Sofia Vasilyevna Kovalevskaya :lol:


#60
AnhTran2911

AnhTran2911

    Thượng sĩ

  • Thành viên
  • 230 Bài viết

Bài toán làm mạnh một bất đẳng thức quen thuộc

 

$\boxed{\textbf{Bài Toán 26}}$ $\text{[Lê Việt Hưng - Mr Cooper]} $ Chứng minh bất đẳng thức sau đúng với mọi a,b,c. 

\[a^2+b^2+c^2 \ge \sum \dfrac{a(b^2+c^2)}{b+c} \ge ab+bc+ca\]

CM vế đầu tiên:

BĐT đã cho tương đương:

$\sum{a^2}\geq a(b+c) - \frac{2abc}{b+c}$

$\Rightarrow\sum{a^2}+ 2abc(\sum\frac{1}{b+c})\geq 2\sum{ab}$

Áp dụng CS ta CM : $\sum{a^2} + \frac{9abc}{a+b+c}\geq 2\sum{ab}$

Đây là Schur phân thức.


        AQ02

                                 






Được gắn nhãn với một hoặc nhiều trong số những từ khóa sau: bất đẳng thức, am-gm, cauchy, bunyakovski, minskovski, schwarz, holder, thcs

1 người đang xem chủ đề

0 thành viên, 1 khách, 0 thành viên ẩn danh